Jump to content

Photo

tìm lim $\lim \frac{{u_{n}}^{2}}{2\ln n}$

- - - - -

  • Please log in to reply
3 replies to this topic

#1
phatsp

phatsp

    Binh nhất

  • Thành viên
  • 25 posts

cho $\left\{\begin{matrix} u_{1}=1\\u_{n+1}=u_{n}+\frac{1}{\sum_{k=1}^{n}u_{k}} \end{matrix}\right.$

tìm $\lim \frac{{u_{n}}^{2}}{2\ln n}$

Edited by phatsp, 02-01-2014 - 15:06.


#2
vutuanhien

vutuanhien

    Thiếu úy

  • ĐHV Toán Cao cấp
  • 691 posts

 

cho $\left\{\begin{matrix} u_{1}=1\\u_{n+1}=u_{n}+\frac{1}{\sum_{k=1}^{n}u_{k}} \end{matrix}\right.$

tìm $\lim \frac{{u_{n}}^{2}}{2\ln n}$

 

Bài này đợi lâu quá mà không thấy ai giải nên mình giải luôn

+)Ta có 

$$\frac{u_{n+1}^2-u_{n}^2}{2\ln (n+1)-2\ln n}=\frac{\frac{2u_{n}}{u_{1}+...+u_{n}}+\frac{1}{(u_{1}+...+u_{n})^2}}{2\ln \frac{n+1}{n}}$$

$$=\frac{\frac{2nu_{n}}{u_{1}+...+u_{n}}+\frac{n}{(u_{1}+...+u_{n})^2}}{2\ln (1+\frac{1}{n})^n}$$

Do đó 

$$\lim_{n \to \infty} \frac{u_{n+1}^2-u_{n}^2}{2\ln (n+1)-2\ln n}=\lim_{n \to \infty} (\frac{nu_{n}}{u_{1}+...+u_{n}}+\frac{n}{2(u_{1}+...+u_{n})^2})$$ (1)

+)Do $u_{n}$ là dãy tăng nên $u_{1}+...+u_{n}> nu_{1}=n$. 

Suy ra $0< \frac{n}{2(u_{1}+...+u_{n})^2}< \frac{1}{2n}$. Từ nguyên lí kẹp ta có ngay

$$\lim_{n \to \infty} \frac{n}{2(u_{1}+...+u_{n})^2}=0$$ (2)

+)Ta lại có

$\frac{(n+1)u_{n+1}-nu_{n}}{u_{n+1}}=n+1-\frac{nu_{n}}{u_{n+1}}=n+1-n(1-\frac{1}{(u_{1}+...+u_{n})u_{n+1}})$

Do đó 

$\lim_{n \to \infty} \frac{(n+1)u_{n+1}-nu_{n}}{u_{n+1}}=n+1-n(1-0)=1$

(dễ dàng chứng minh được $u_{1}+...+u_{n}$ tiến tới vô cùng  ;))

Suy ra theo bổ đề Stolz, ta có ngay

$$\lim_{n \to \infty} \frac{nu_{n}}{u_{1}+...+u_{n}}=1$$ (3)

Từ (1), (2), (3) ta có

$\lim_{n \to \infty} \frac{u_{n+1}^2-u_{n}^2}{2\ln (n+1)-2\ln n}=1$

Do đó theo bổ đề Stolz, ta có

$$\lim_{n \to \infty} \frac{u_{n}^2}{2 \ln n}=1$$

(đpcm)  :D


Edited by vutuanhien, 22-01-2014 - 11:51.

"Algebra is the offer made by the devil to the mathematician. The devil says: I will give you this powerful machine, it will answer any question you like. All you need to do is give me your soul: give up geometry and you will have this marvelous machine." (M. Atiyah)

 


#3
phatsp

phatsp

    Binh nhất

  • Thành viên
  • 25 posts

+)Ta lại có

 

$\frac{(n+1)u_{n+1}-nu_{n}}{u_{n+1}}=n+1-\frac{nu_{n}}{u_{n+1}}=n+1-n(1-\frac{1}{(u_{1}+...+u_{n})u_{n+1}})$

Do đó 

$\lim_{n \to \infty} \frac{(n+1)u_{n+1}-nu_{n}}{u_{n+1}}=n+1-n(1-0)=1$

(dễ dàng chứng minh được $u_{1}+...+u_{n}$ tiến tới vô cùng  ;))

Khúc này mình thấy kì kì,nếu mà $\frac{1}{(u_{1}+...+u_{n})u_{n+1}}$ tiến tới 0 còn n tiến đến vô cùng thì là vô định chứ sao tính lim rồi mà vẫn còn n+1-n(1-0) ở ngoài .Nếu co sai gì thi mình xin lỗi



#4
phatsp

phatsp

    Binh nhất

  • Thành viên
  • 25 posts
à hiểu rồi khúc đó phải là $\lim_{n \to \infty} \frac{(n+1)u_{n+1}-nu_{n}}{u_{n+1}}=n+1-n(1-0)=1$,xin lỗi không sai gì cả

Edited by phatsp, 25-03-2014 - 23:35.





1 user(s) are reading this topic

0 members, 1 guests, 0 anonymous users